Friday, October 21, 2011

Magic versus metaphysics

Any sufficiently advanced technology is indistinguishable from magic.

Arthur C. Clarke

Any sufficiently rigorously defined magic is indistinguishable from technology.

Larry Niven

Some atheists are intellectually serious.  Some are not.  There are several infallible marks by which an atheist might show himself to be intellectually unserious.  Thinking “What caused God?” is a good objection to the cosmological argument is one.  Being impressed by the “one god further” objection is another.  A third is the suggestion that theism entails a belief in “magical beings.”  Anyone who says this either doesn’t know what theism is or doesn’t know what magic is.  Or (no less likely) doesn’t much care one way or the other – it’s another handy straw man, useful for those who want to believe that theistic arguments are manifestly fallacious or otherwise silly, or who find it rhetorically useful to pretend that they are.

What is magic?  In Renewing Philosophy, Hilary Putnam makes some interesting remarks on the subject:

If a witch must have magical powers, then it is far from clear that the concept of a witch is a coherent one, because it is far from clear that the concept of a magical power is a coherent one.  We can certainly imagine possible worlds in which things regularly happen that superstitious people would regard as magic; but the very fact that they regularly happen in those possible worlds is strong reason for saying that in those possible worlds those things are not really magic—it is just that those worlds have different laws than the actual world.  The notion of a world in which things happen that are “truly magical” is, I think, an incoherent one; and that means, I think, that the notion of a witch is an incoherent one.

One might try to meet this difficulty by defining a witch not as someone who has magical powers but as someone who has supernatural powers, where the supernatural is understood not in terms of the notion of magic, but in terms of not falling within the categories of substance, space, and time.  It is extremely doubtful that the pagan witches, or the witches of present-day African tribes, are supposed to derive their powers from something which is supernatural in that sense.  It is a feature, in fact, of pagan thought that the gods, demons, and so on, are not supernatural in the sense which came into existence with the rise of Greek philosophy and the incorporation into the Jerusalem-based religions of a certain amount of Greek philosophy.  The notion that what is magical must derive from the supernatural, in the philosophical/theological sense of “supernatural”, is not part of the original meaning of the term. (p. 44)

Putnam surely captures one important sense of the term “magical” here (though there are other senses, as we will note below).  More to the point, he surely captures the sense of “magical” in which the notion of magic is thought by the atheist to be objectionable.  And rightly so, for it is objectionable.  “Magical” powers, as Putnam here describes them, are powers which are intrinsically unintelligible.  It’s not just that we don’t know how magic operates; it’s that there is, objectively, no rhyme or reason whatsoever to how it operates. 
 
That it is intrinsically unintelligible has to be what is objectionable about it.  For it is not reasonable to object to the notion of powers or causes which are intelligible in themselves, but which we simply don’t happen to understand, or perhaps even cannot understand given the limitations on our intellects.  There is, after all, no reason to think that whatever exists simply must be comprehensible to us -- especially for someone who regards our cognitive powers as the product of evolutionary processes that favor survival value rather than accurate beliefs per se.  Indeed, some naturalists have insisted that there are limits in principle to what we can understand, so that certain aspects of the natural world must remain forever mysterious to us.  There can be serious arguments for the postulation of such limits on our knowledge, and such a postulation can do real explanatory work -- again, for the naturalist or atheist no less than for the theist.  (In an earlier post, I discussed the various senses in which different aspects of the world might be said to be intelligible or unintelligible, from either an atheist point of view or a theistic one.)

So, again, what is objectionable about magic can only be that it is supposed to be inherently unintelligible, unintelligible even in principle and not merely in practice.  Appeals to magic in this sense can, of necessity, explain nothing.  They are rightly dismissed as pseudo-explanations or worse -- Putnam suggests that they are actually incoherent.  (He does not elaborate, but perhaps his point is that it is incoherent to suppose that an appeal to “magic” is any kind of explanation given that an explanation necessarily makes the explanandum intelligible, and the notion of magic is the notion of that which is inherently unintelligible.)

But the greatest theistic writers -- thinkers like Aristotle, Plotinus, Aquinas, Leibniz, and the like -- would agree that the notion of “magic” in this sense is intellectually disreputable.  And when they argue for the existence of God, they are not appealing to magic.  On the contrary, they are appealing precisely to rational considerations about what the world must be like in order to be intelligible.  For example, the Aristotelian-Thomistic (A-T) argument from motion rests on the distinction between actuality and potentiality (or “act and potency”), which was introduced by Aristotle as necessary to explain how change is possible.  Thinkers like Parmenides and Zeno had argued that change is not possible.  Their reasons are complex, but they involve the crucial assumption that a thing either has being or existence full stop, or it is sheer nothingness or non-being.  But change, they argued, would have to involve going from non-being to being, and thus from nothing to something; and from nothing, nothing can come.  Hence change is impossible, and the appearance of change illusory.

Aristotle argued that the assumption such arguments rest on is false, for in addition to that which is utterly non-existent on the one hand, and that which is fully real or actual on the other, we have to distinguish a third, middle ground category of what exists potentially.  A certain rubber ball might on the one hand be actually red, actually spherical, actually solid, and actually smooth to the touch; and on the other hand, it would be in no way a rabbit, or a quasar, or a Buick Skylark.  But in between these extremes there are the ways it potentially is, just given its nature -- for instance, it is potentially green (if you paint it), potentially flat and squishy (if you melt it) and so forth.  And that is how change is possible -- it does not involve going from sheer non-being to something actual (which would be impossible), but rather going from potentiality to actuality.  For while a potential is not actual, it is not nothing either.

But a potential, precisely because it is merely potential and not actual, cannot actualize itself; only what is already actual can actualize it.  And if that which actualizes a potential is itself being actualized as it does so, it must in turn be actualized by something else.  Such a regress of causes would be of the essentially ordered or instrumental kind; and it can only terminate (so the A-T philosopher argues) in that which can actualize without itself having to be actualized -- something which just is “pure actuality.”   And that is the metaphysical core of the A-T conception of God.

Now, there is more to the story than that.  The point for now, though, is not to develop or defend this sort of argument.   (I have done so elsewhere, e.g. here, here, and here.)  The point is rather to emphasize that there is nothing remotely “magical” about it.  You might disagree with the argument; you might think (quite wrongly, I would say, but let that pass) that it has somehow been superseded by modern science, or that in some other way it is fallacious or rests on mistaken premises.  What you cannot reasonably do is deny that such an argument is a genuine attempt at explanation, rather than an appeal to something inherently unintelligible.  The same can be said of the Thomistic argument from the distinction between a contingent thing’s essence and its existence to God as a cause whose essence just is existence; or the Neo-Platonic argument from the existence of multiplicity to a cause which is an absolute unity; or the Leibnizian argument from contingency to a necessary being; or indeed of any of the other major theistic arguments.  It is one thing to reject these arguments after a serious analysis of them.  But to dismiss them as appeals to “magic” is just silly.

Notice that Putnam rightly distinguishes the “magical” from the “supernatural.”  As I have noted before, “supernatural” does not have, in traditional theology, the connotations that movies, television, and the like have given it in the popular mind.  In particular, it does not have any necessary connection with belief in ghosts or other paranormal phenomena.  The “supernatural” is just that which transcends the natural order.  And if it is not governed by the laws that govern the natural order, that is not because it is less intelligible than the natural order, but because it is more intelligible, and indeed the source of the intelligibility of the natural order.  The natural order is contingent; its divine, supernatural ground is necessary.   The causal processes in terms of which we explain everyday happenings within the natural order are secondary, having only a derived efficacy; the divine, supernatural first cause is that which has its causal power inherently, in an absolutely underived way.  (See again the post on essentially ordered or instrumental causes linked to above.)  And so forth.  

Again, even if this whole picture were rejected as outdated metaphysics, that does not entail that it is “magical.”  Outdated scientific theories which appealed to notions like phlogiston, caloric, celestial spheres and the like were not “magical”; they were mistaken, but they were not appeals to what is intrinsically unintelligible.  Similarly, even if it turned out that Aristotelian metaphysics, Platonic metaphysics, Thomistic metaphysics, and Leibnizian metaphysics were all mistaken, that would not make them appeals to “magic.”  

Nor will it do to insist that only scientific or naturalistic explanations could even in principle be non-magical.  For one thing, such a claim would presuppose something like a verificationist theory of meaning, insofar as it implies that non-naturalistic or non-scientific explanations are not even intelligible; and semantic verificationism is self-defeating.   For another thing, scientism and naturalism are themselves self-defeating (unless they are merely trivially true), and tend to rest on non sequiturs -- that is, when they are actually being argued for at all, as opposed to being merely asserted.  (I’ve discussed these problems here, here, here and here.)

Indeed, if any view is plausibly accused of being “magical” in the sense in question, it is atheism itself.  The reason is that it is very likely that an atheist has to hold that the operation of at least the fundamental laws that govern the universe is an “unintelligible brute fact”; as I have noted before, that was precisely the view taken by J. L. Mackie and Bertrand Russell.  The reason an atheist (arguably) has to hold this is that to allow that the world is not ultimately a brute fact -- that it is intelligible through and through -- seems to entail that there is some level of reality which is radically non-contingent or necessary in an absolute sense.  And that would in turn be to allow (so the traditional metaphysician will argue) that there is something which, as the Thomist would put it, is pure actuality and ipsum esse subsistens or “subsistent being itself” -- and thus something which has the divine attributes which inexorably flow from being pure actuality and ipsum esse subsistens.  Hence it would be to give up atheism.

But to operate in a way that is ultimately unintelligible in principle -- as the atheist arguably has to say the fundamental laws of nature do, insofar as he has to say that they are “just there” as a brute fact, something that could have been otherwise but happens to exist anyway, with no explanation -- just is to be “magical” in the objectionable sense.  In fact it is only on a theistic view of the world that the laws of nature are not “magical”; and the Mackie/Russell position is (as I argue in the post linked to above) ultimately incoherent for the same sorts of reason that magical thinking in general is incoherent.  As is so often the case, the loudmouth New Atheist turns out to be exactly what he claims to despise -- in this case, a believer in “magical powers.” 

Of course, there are other senses of the word “magic.”  For example, the term is also used to refer to phenomena that are paranormal or occult, but not intrinsically unintelligible -- phenomena which do have an explanation, but where the explanation lies beyond the everyday material order of things and is to a significant extent closed to our investigation.  Now, as I indicated earlier, there is no necessary connection between the “supernatural” (in the theological sense) and the “magical” in this paranormal sense.  Someone could be a theist and reject all alleged paranormal phenomena.  And someone could be an atheist and believe that there are some genuine paranormal phenomena.  (C. D. Broad was one example of such an atheist.  I do not know whether Stephen Braude would call himself an atheist, but his interest in the paranormal does not seem to be motivated by any religious concern.)

To be sure, many theists do in fact believe in paranormal phenomena.  Alleged paranormal practices of the sort often labeled “occult” or “magical” are condemned by the Catholic Church, not merely because they are often phony (though of course they often are phony), but because even if authentic they involve an appeal to demonic powers or lost souls.  Now angels, demons, and souls are of course associated in the popular mind with all sorts of superstitions and crude images.  But rightly understood there is nothing superstitious about them, and certainly nothing “magical” in the objectionable sense of being intrinsically unintelligible.  The traditional philosophical arguments for the immateriality of the intellect provide independent grounds for holding that it is possible in principle for there to be a disembodied intelligence.  And in traditional theology, that is exactly what an angel, a demon, or a postmortem soul is supposed to be.  Here too, while one could of course disagree with the arguments in question, they are not “magical” in the sense of appealing to powers regarded as intrinsically unintelligible.  (It is worth emphasizing that Aristotle himself, who had no Christian theological ax to grind, thought that there were such things as disembodied intelligences.)  

The term “magic” is also sometimes used ironically -- for something that is so contrary to ordinary experience and existing knowledge as to seem unintelligible, but which is in fact perfectly intelligible in itself and can be made intelligible to us given sufficient advances in our knowledge (and which is thus not really “magical” at all).  This seems to be the sense in which Arthur C. Clarke and Larry Niven use the term in the statements quoted above.  And in this ironic sense, theological claims may well be “magical,” at least to those ignorant of what serious theologians and philosophers of religion have actually said -- just as scientific claims would seem magical to those unacquainted with modern science.  This suggests the need for a third law to supplement Clarke’s Law and Niven’s Law:

Any theological proposition will seem “magical” to someone insufficiently versed in the underlying metaphysics. 

278 comments:

  1. E.H. Munro,

    "Sexiness is a subjective valuation made by an observer. I suppose that all matter has the potential for sexiness, but only insofar as conscious observers so evaluate it."

    How is it more subjective than 'final cause' or 'intentionality'? Intentionality itself is an exercise in subjectivity. Final cause can also be seen as a subjective choice as to what the purpose of a thing is.

    ReplyDelete
  2. djindra quotes and comments.

    " 'Empiricism is a theory of knowledge, not an account of reality or what exists.'

    True, but if we can't know about it how are we to have any knowledge that it exists?

    October 27, 2011 9:36 AM"


    How are we to "have knowledge that it exists"?

    That would depend on whether your proposed reliance on a strict empiricism as a complete or sufficient criterion for having any knowledge was misplaced or not, wouldn't it.



    And thus, we come full circle and wrap up, eh?

    ReplyDelete
  3. Dguller: Well, if we knew the causes, then we wouldn’t be ignorant, and thus wouldn’t need to postulate uncaused events at all.

    But we don't need to postulate uncaused events! We have (Aristotelian/Scholastic) philosophies that explain reality without doing so, so one must be pretty desperate to refuse them and pick a "system" that's unreasonable instead.

    I think that metaphysical arguments rely upon premises that inevitably generalize from our empirical experience,

    They depend upon reason. Intuition may give us some ideas to get going, but in the end the argument doesn't depend on it, or it isn't a proper metaphysical argument at all. That is, metaphysics will say things like "If something comes into being, it must be caused." Now, we need some experience to apply this proposition. ("X came into being. Therefore X was caused.") However, the conditional itself does not depend on our experience of any X. In that sense, metaphysics does not depend on our questionable experiences. The principle remains even if there were no Xs anywhere.

    QM wouldn’t work if 1+1=3. The math wouldn’t add up, and the theory would fall apart.

    How can you say the math wouldn't add up? If 1+1=3, maybe 3+3=4? But seriously, of course you're right, the theory would fall apart, and so it does with uncaused events. If these things really were uncaused, then the stats wouldn't add up. But they do.

    First, mathematics has done the impossible in the past. It allowed irrational numbers, imaginary numbers, and so on, and thus the rules had to be revised on a number of occasion, showing that the previous understanding was false, or at least, limited.

    Yes, and the reason we could show that our past intuitions or experiences were wrong is because we followed logic instead. Those things were never really impossible, but that wasn't clear until we pushed the reasoning through. Saying, "maybe QM has uncaused events because I sure can't imagine how it works" is like saying "maybe negative numbers have no square roots because I can't imagine how that works". Except in this case, the logic has been worked out centuries ago, so all we have to do is read up on it.

    I think it is possible that there are aspects of reality that simply cannot be captured by logic and reason. Maybe all of reality is rational, or maybe it isn’t. The only way we will ever know is when all of reality is explained according to logic and reason. Until then, it is an operational assumption that has been highly successful thus far.

    It's not merely an "operational assumption", because you can't just toss it aside and try some alternative instead! So even if you were right, there would still be no reason to abandon logic because we can't think any other way. (Well, we can, but only by doing it badly!) The only way we will ever find out if all reality is explicable according to logic and reason is to pursue rational systems to their limit. So even in that vein, one ought to pragmatically follow Scholastic philosophy (or similar) because it so far is able to explain reality entirely according to logic and reason.

    [Bell's inequality means reality] is either (a) non-local, but involving hidden variables or (b) local, but lacking hidden variables.
    Both (a) [instantaneous connections] and (b) [inherently uncaused] are problematic, and require giving up key intuitions that we have about how reality works:


    Actually, I think our intuitions are more likely go for non-locality. Localism is not an intuition but rather a habit instilled by studying physics (including non-intuitive Relativity). But being prepared to give up our intuitions does not mean we should give up reason as well. Keep in mind that we haven't found problems with metaphysics, just with one particular proposed system: Billiard-Ball metaphysics. So we should reject Billiardballism, but not metaphysics per se.

    ReplyDelete
  4. DNW - They have a public meaning not dependent on the decision or interpretation of a potential poacher.

    Don't you mean they have a consensual meaning, one that we as English speakers/writers have agreed on? Is there an objective, necessary reason that the letter 'n' signifies "dental or alveolar nasal", or that the syllable "no" indicates negation?

    You cannot read hieroglyphics and the old kingdom no longer exists. Does this mean the obelisk has no objective meaning?

    It had meaning to the people who wrote it - they had their agreement over what the different marks signified. There was no objective reason why those particular marks meant those particular things, however. And as you've noted, they would be meaningless to me.

    The "smiley-face" symbol has at least a vague agreed-upon meaning. Does this have an objective meaning?

    You are hiking and realize you are being stalked by a brown bear. There are undeniable practical implications to this for any potential prey, as well as whatever subjective interpretations may occur to the target.

    There are "undeniable practical implications" of a boulder rolling down the trail toward a hiker, too. Does the boulder mean something by it, though?

    Besides, "undeniable practical implications" aren't the same as the 'meaning' we're talking about (such as 'the meaning of life'). What if the hiker's a nature-worshipping terminal cancer patient who's decided to sacrifice themselves to a bear? What's the meaning of that event then?

    "...if by objective meaning you refer to an event's or object's meaning independent of the inclinations or capacities of one particular mind or subject, then of course it is coherent to speak of an objective meaning."

    Sure, in the sense of a group consensus, as I've noted above. But that's not the same as a meaning that is "independent of the inclinations or capacities of" any or all minds.

    ReplyDelete
  5. DNW - "What you are actually saying is that all purposes are linked to minds. So what?"

    Well, atheists may not believe 'the mind of God' but they believe in at least human minds, right?

    So what the heck were you claiming when you said "They are just another pointless pattern"? Why couldn't atheists see a point to things like "the awesomeness of sentience", or "the grandeur of... "life""?

    I mean, if there were no minds, there wouldn't be any point. But since there are minds, there can be lots of points and significance and meaning.

    ReplyDelete
  6. DNW - "It’s an experience, an emotion, not an assertion of fact."

    I don't know anyone serious who would take such stuff at face value. You don't really, do you?


    When talking about finding meaning in the lives of oneself and others, yeah it pretty much is. How would you dispute it if someone claimed they found significance in their life? In your life?

    My wife and kids mean an enormous amount to me - more than I mean to myself. I can assert the fact of the meaning, but what about the meaning itself?

    ReplyDelete
  7. Grodrigues:

    Honestly, I do not know what else to add. I probably am going to be unfair, but all I am reading in your complaints is a personal argument from incredulity. I would also wager that you are conflating imagining and conceiving via the intellect. We may not be able to form a clear image of non-being, quite plausibly so, because all we ever sense is being and never non-being, but we can quite clearly form the concept of non-being. It is actually a particularly simple concept. Lacking in all potentiality, there is not much that can be said about it, is there? But we *can* think and talk meaningfully about the concept. For example, is it instantiated, meaning does non-being exist? Of course not, because if non-being existed then it would be actual (in admixture with potentiality or pure act) but since it is not actual in any way, it does not exist -- at least in this specific sense.

    The concept of non-Being inherently involves Being by virtue of being negated. As such, whenever we use the concept of Nothing, we are automatically using the concept of Something, and thus never actually have a pure concept of non-Being without any admixture of Being. Again, it would be like saying that one can avoid thinking about a pink elephant while entertaining the thought that there is no pink elephant. The other analogy that I mentioned was walking along a clean floor in muddy shoes but insisting that the floor is still clean. It does not seem to be possible, because the negation necessarily involves the affirmation.

    Maybe our intuitions just vary on this matter, and I’m happy to leave it at that, but I do wonder how you can think “not-X” without simultaneously thinking about “X”.

    ReplyDelete
  8. DNW,

    For an empiricist to know something exists "would depend on whether your proposed reliance on a strict empiricism as a complete or sufficient criterion for having any knowledge was misplaced or not, wouldn't it?"

    Yes. We cannot go through the day without making certain assumptions. I do assume that if I grab a rock, and can feel it, and can throw it towards you and you duck, that the rock does exist. The question, IMO, finally comes down to feedback. Do the results reached through the assumptions positively reinforce the assumptions or not? How thoroughly do they do so? Are there extra assumptions that ultimately have no predictive power and end up being extraneous? Sometimes going around in full circle is not a bad thing.

    ReplyDelete
  9. dguller:

    I don't see why would even want a "pure" concept of non-being. Unless the concept of non-being has something to do with being, it's not worth "knowing".

    ReplyDelete
  10. dguller,

    I do wonder how you can think “not-X” without simultaneously thinking about “X”.

    You can't. Let me try this again. In any judgment, any proposition you can come up with, both terms are always implicit due to the principle of non-contradiction. You can't get around this if you want to reason.

    Proposition:

    Dguller is sitting at the computer right now.

    Implied in this judgment is the acceptance of X or not-X. And you understand completely what either X or not-X means. Appeals to agnosticism won't help, because every meaningful proposition has a truth value by nature, whether we know the answer or not.

    Essentially, your position on the "pollution" of a concept of non-Being leads to an outright denial of Reason. It is to say that we can never know 'not-X' anything.

    ReplyDelete
  11. How is it more subjective than 'final cause' or 'intentionality'? Intentionality itself is an exercise in subjectivity. Final cause can also be seen as a subjective choice as to what the purpose of a thing is.

    Evolution occurs whether or no anyone observes it, "sexiness" only occurs in the mind of the observer.

    ReplyDelete
  12. Ray,

    I think it was good that you clarified your question before I replied, since I think it reveals that you're conflating "knowing that X" and "knowing that X is knowable", and I think that it comes from your method. Replying to Fermat's Last Theorem, you seem to be asking that even though we have a supposed proof of it, how do we know that there isn't an error in it and that it isn't actually wrong and, therefore, unknown? And I, of course, concede that -- although I would reply that knowledge does not require absolute certainty, and so some doubt does not in any way mean that we wouldn't know it.

    But I would also reply that this is irrelevant to considerations of knowability, and as proof point to my method for determining, at least, unknowability. Nothing in my method says that after you've determined it to be knowable that you, in fact, actually know it. We could, in theory, determine if FLT is knowable in principle even if we may not ever know it in practice.

    In contrast, your main method for determining if something is knowable is to try to know it, and if you do then it's knowable. But this leads you precisely to the problems that you're having. In the first place, if you fail to come to know it, you do raise the question of whether it's actually unknowable or whether you just haven't done that yet. And then you also run into the issue that if you DO come to know it, if that status ever changes you're right back to wondering if it's unknowable.

    But I think that this is the wrong way to go about trying to determine knowability. Knowability and knowing are two different questions, and while you can answer the two simultaneously I'd argue that trying to know something implies at least a belief that it is knowable, and if that belief is not at all justified before you try to come to know it then your whole procedure is a waste. As long as you're willing to presume knowability to try to know it, why care about the knowability question at all? If you want to know if something is knowable or not, stop trying to know it and start working on the concept to see if the concept itself implies knowability or unknowability ... which may well give hints on how to come to know it.

    ReplyDelete
  13. @DNW

    I think the article that Ray linked to is suggesting (and doing a very poor job at that) that subjective "meaning" is meaningless. This may be a retreading of the tired old analytic/synthetic distinction of logical positivism. Why anyone would take this seriously any longer after Popper, Quine, and Wittgenstein is beyond me.

    Logical positivism is dead. The only people who believe in it anymore are scientists who want to turn their method into something that can explain all of reality and the new atheists, who are nearly all philosophically illiterate.

    ReplyDelete
  14. Mr. Green:

    But we don't need to postulate uncaused events! We have (Aristotelian/Scholastic) philosophies that explain reality without doing so, so one must be pretty desperate to refuse them and pick a "system" that's unreasonable instead.

    Ultimately, there is no argument to be made about this issue, because it is about differing intuitions. My intuition is that beyond our understanding, we simply do not know what is happening. In that case, either what is happening is similar to what we know, or is different from what we know. The bottom line is that until it is subsumed under our understanding, we just don’t know. That means that we should all just take it easy, and admit that we have no idea what we are talking about.

    They depend upon reason. Intuition may give us some ideas to get going, but in the end the argument doesn't depend on it, or it isn't a proper metaphysical argument at all. That is, metaphysics will say things like "If something comes into being, it must be caused." Now, we need some experience to apply this proposition. ("X came into being. Therefore X was caused.") However, the conditional itself does not depend on our experience of any X. In that sense, metaphysics does not depend on our questionable experiences. The principle remains even if there were no Xs anywhere.

    And where does reason get its principles? Do they automatically turn on as soon as reason is mature? Where do those principles come from? How do we know that they are reliable?

    My understanding is that even Aristotle recognized that universal principles require the experience of particular events, and abstracting the general forms of those particular events. It is not as if reason already contained all these general forms a la Plato’s Meno, but rather that its interaction with particular events in the world involves a transaction of sorts.

    Yes, and the reason we could show that our past intuitions or experiences were wrong is because we followed logic instead. Those things were never really impossible, but that wasn't clear until we pushed the reasoning through. Saying, "maybe QM has uncaused events because I sure can't imagine how it works" is like saying "maybe negative numbers have no square roots because I can't imagine how that works". Except in this case, the logic has been worked out centuries ago, so all we have to do is read up on it.

    We did not follow logic in those cases. With regards to imaginary numbers, a conscious choice was made to violate rules that were considered inviolable, and it turned out to be fruitful from a theoretical and practical matter. My point is that if mathematicians, for example, are allowed to do the impossible, which automatically changes what is possible to begin with, then it has nothing to do with logic. It has more to do with taste and choice rather than any logical inferences. Perhaps you can explain the logical rationale that justifies the existence of the square root of -1?

    ReplyDelete
  15. Mr. Green:

    Yes, and the reason we could show that our past intuitions or experiences were wrong is because we followed logic instead. Those things were never really impossible, but that wasn't clear until we pushed the reasoning through. Saying, "maybe QM has uncaused events because I sure can't imagine how it works" is like saying "maybe negative numbers have no square roots because I can't imagine how that works". Except in this case, the logic has been worked out centuries ago, so all we have to do is read up on it.

    We did not follow logic in those cases. With regards to imaginary numbers, a conscious choice was made to violate rules that were considered inviolable, and it turned out to be fruitful from a theoretical and practical matter. My point is that if mathematicians, for example, are allowed to do the impossible, which automatically changes what is possible to begin with, then it has nothing to do with logic. It has more to do with taste and choice rather than any logical inferences. Perhaps you can explain the logical rationale that justifies the existence of the square root of -1?

    It's not merely an "operational assumption", because you can't just toss it aside and try some alternative instead! So even if you were right, there would still be no reason to abandon logic because we can't think any other way. (Well, we can, but only by doing it badly!) The only way we will ever find out if all reality is explicable according to logic and reason is to pursue rational systems to their limit. So even in that vein, one ought to pragmatically follow Scholastic philosophy (or similar) because it so far is able to explain reality entirely according to logic and reason.

    It is still an assumption that is unproven. And I am not saying to abandon logic and reason. Actually, I’m saying to, as you put it, “pursue rational systems to their limit”, because that’s the only way to determine what their limits are. I find it hard to believe that logic is unlimited and our brains are capable of determining the truth about all aspects of reality. We sure might, but that is an assumption that has not been demonstrated. I don’t know how you can go from “we understand many things” to “we can understand everything”. Perhaps you can fill in the missing part of the argument?

    And can Scholastic philosophy explain the exact point where one species becomes another? And if it cannot, then what does that say about the doctrine of essence or nature?

    Actually, I think our intuitions are more likely go for non-locality. Localism is not an intuition but rather a habit instilled by studying physics (including non-intuitive Relativity). But being prepared to give up our intuitions does not mean we should give up reason as well. Keep in mind that we haven't found problems with metaphysics, just with one particular proposed system: Billiard-Ball metaphysics. So we should reject Billiardballism, but not metaphysics per se.

    I thought the issue with localism is that causes must precede effects, and that there must be a series of mediating steps between the cause and the effect. Whether those intermediary steps involve billiard ball contact or forces, for example, is irrelevant. But anyway, intuitions vary about this issue.

    ReplyDelete
  16. @dguller:

    "When Bell’s inequality talks about hidden variables, it is referring to properties of quantum particles that are objectively present, but unable to be determined by us, which is why they are hidden, but not non-existent. It seems that you are saying that your hidden variable (or “natural cause”) is not a variable at all, because it cannot be represented by mathematics. So, when Bell’s inequality shows that combining locality with hidden variables is impossible, you are avoiding this by denying that your “natural cause” is a variable at all! In other words, you are taking the epistemological randomness option, but saying that there are truths that cannot be specified by mathematical formalism, and thus avoid Bell’s inequality."

    If one wants to be pedantic, not being a variable means that it cannot be described as a random sample drawn from a probability space with a well-defined probability and expectation value. But yes, you are basically correct. Just a couple of comments. First, my proposal is not meant to exaust all possibilities; the point to stress is probably more the fact that science, in the wider sense, is not reduced to the mathematization of observed correlations. The main aim of science is understand, to make intelligible. We should remind ourselves that from the level of biology upwards, mathematics has little to no more than a nominal presence. By the way, have you followed the link to the Telic Thoughts blog that someone provided some posts back? Some good stuff there. Second, as I said previously, I tend to favour non-locality, because in my view that is what QM seems to be telling us. But also as I said, non-locality opens some very serious problems, so this is just an inclination not a final judgement.

    "My point was that if one can accept that mathematics has limitations in terms of understanding the world, then why the reluctance when it comes to logic and reason?"

    You would have a point if rationality is reducible to mathematics. As I said above, from biology upwards, there is very little mathematization going on but there is still an impressive shared fund of accumulated knowledge.

    "I think that you are right. My own view is that I know that much of the world is intelligible, and that the rest might be equally intelligible, but I really do not know if this is true. It is a guiding assumption that has worked out very well in a number of respects, but only time and work will tell if it is true."

    It is not and cannot be a "guiding assumption", because it is the very presupposition for knowing the world in the first place! Your musings puzzle me because my intuitions lead me the opposite way. Although I have no illusions about human limitations, the fact is that I cannot even imagine what an uncaused event would look like. Is it contingent? then it must have a cause. To quote from the great mathematician D. Hilbert: "Wir mussen wissen. Wir werden wissen." or "We must know. We shall know."

    ReplyDelete
  17. JA said...

    @DNW

    I think the article that Ray linked to is suggesting (and doing a very poor job at that) that subjective "meaning" is meaningless. This may be a retreading of the tired old analytic/synthetic distinction of logical positivism. Why anyone would take this seriously any longer after Popper, Quine, and Wittgenstein is beyond me.

    Logical positivism is dead. The only people who believe in it anymore are scientists who want to turn their method into something that can explain all of reality and the new atheists, who are nearly all philosophically illiterate.
    October 27, 2011 1:19 PM "


    JA,

    I'm not sure what proposition exactly, is being advanced. Ingles' link seemed to me to be arguing that meaning is a synonym for the subjective experience of an emotion.


    What we are supposed to make of that, I don't know. There is not really any argument for it. Just an assertion that people don't understand what "meaning" really means because they are accustomed to thinking in terms of a kind of uncritical shorthand which has led them astray. Reading it is kind of like listening to some kid grandly pointing out to you that the words "why" and how" actually have different meanings, and that "why" when properly used, assumes some level of intent, and that therefore we should now restrict ourselves to asking how the car is not working rather than why.

    I would however agree with the first sentiment expressed in the essay. The one that suggest that the kind of pseudo-religious babble PZ Myer's was engaging in is enough to make you cringe.

    But I cannot see that the author goes on to make much of a case for anything at all with the rest of his "argument".

    ReplyDelete
  18. JA - "I think the article that Ray linked to is suggesting (and doing a very poor job at that) that subjective "meaning" is meaningless."

    No, exactly the opposite, and it's rather amazing to me that you could misread it so.

    ReplyDelete
  19. @DNW:

    Let me rephrase and extend what I meant. The argument is a variant of the ethical theory of emotivism first posited by the logical positivism of A.J. Ayer. If a claim is neither synthetic (tied to an empirical observation) nor analytic (tautological) than it is subjective and carries no content that is objectively verifiable; it is nothing more than an expression of emotion.

    Since the author identified "meaning" as something that can only reference a subjective emotion, rather than something objective or tautological, then it has no basis for public discourse--hence his drivel about personal meaning and existentialism.

    Again, no serious person really argues this anymore after Quine, Wittgenstein, and Popper. Alasdair MacIntyre, an analytical Thomist, does an excellent job of both refuting emotivism and situating it as a moral pathology that developed due to historical circumstance in "After Virtue."

    But something else also needs to be acknowledged: Emotivism assumes the fact-value distinction, which Aristotelians reject outright. What is going on, is that Ray is talking about meaning with different ontological and epistemological assumptions operating in the background of his thought than you are. Any dialogue with him needs to take this into account.

    ReplyDelete
  20. @Ray:

    It shouldn't be. The article was rather incoherent. No serious philosopher makes claims like that anymore.

    ReplyDelete
  21. I think dguller is confusing the concept of nothing, which is something, with nothing. Nothing "is" just nothing.

    ReplyDelete
  22. Ray Ingles writes,

    "'DNW - They have a public meaning not dependent on the decision or interpretation of a potential poacher.'

    Don't you mean they have a consensual meaning, one that we as English speakers/writers have agreed on?"

    No I don't mean that. I mean that the subjective disposition of the poacher and his moods and inclinations are irrelevant to the meaning of the English words that constitute the sense of the sign.

    You seem to have confused your own issue here by importing the term "consensual", i.e., by mutual agreement, into your theory of meaning; when the term "conventional" as applied to the use and application of an arbitrary sign for a sound, is what could more plausibly be argued.

    In addition I cannot see how your theory of consensual meaning, comports with your assertions of meaning's radically subjective nature. It instead seems to contradict it.

    In any event, we were originally talking of objectively ascertainable meanings. And by objective I meant that the meaning of the no trespassing sign is not determined by the inclinations or internal emotional disposition of the poacher, as your subjective theory of meaning would suggest.

    Now, if you wish to argue against intrinsic meaning or "a point" inherent to some natural material object per se, go right ahead. If you do so you will be granting my original characterization of the metaphysical status of the concept of meaning according to the views of the people to whom blogger Machinephilosophy was adverting.

    cut ...

    ReplyDelete
  23. Cont ...

    Ray Ingles asks ...


    " Is there an objective, necessary reason that the letter 'n' signifies "dental or alveolar nasal", or that the syllable "no" indicates negation?"


    I would like to know why you have now introduced the term "necessary" here as regards an objective determination.


    I have so far made no objection to what look like deliberate shifts in your footing. But if you are going to switch from arguing over whether my use of the term "pointless pattern" was a logically legitimate characterization of the atheist viewpoint from an old transcosmic perspective, to instead requesting a demonstration of the "objective, necess[ity?]" that a particular "convention" should be applied to a particular phenomenon, you might show how it pertains to what has actually been stated or argued.

    Didn't you even glance at the dictionary definition of the term "objective" as I implied you ought? It might help cut to the chase.

    " 'You cannot read hieroglyphics and the old kingdom no longer exists. Does this mean the obelisk has no objective meaning?'

    It had meaning to the people who wrote it - they had their agreement over what the different marks signified. There was no objective reason why those particular marks meant those particular things, however. And as you've noted, they would be meaningless to me."

    Again, you seem to be having difficulties understanding the term "objective".

    The fact that it would be meaningless to you would be a reflection of your relative ignorance, not upon the lack of meaning to the text. An Egyptologist would be quite well able to tell you what the signs actually meant, and he would not have to rely on whether he felt slouchy or inspired or awed or upon any other mood, for his rendering of the sense.

    The attachment of a particular sign to a particular referent - especially in an alphabetic script - is not what determines whether the sign has an objectively ascertainable meaning.

    Again, you seem to be taking objective to mean something like intrinsic. Though what intrinsic could possibly mean under your system of interpretation, is anyone's guess.


    Anyway, the observation that conventional signs differ while meanings are intersubjectively determinable is nothing new.

    Certainly you are familiar with this text?


    "Spoken words are the symbols of mental experience and written words are the symbols of spoken words. Just as all men have not the same writing, so all men
    have not the same speech sounds, but the mental experiences, which these directly symbolize, are the same for all, as also are those things of which our experiences are the images."


    That was from "On Interpretation", by Aristotle. E. M. Edghill trans.

    [Of course, you may wish to do away with the concept of man as having anything but a subjectine emotion based "meaning". If you do, just say so.]


    And certainly you do not deny that propositions exist or have objectively ascertainable meanings?

    However, a widely employed and authoritative definition of "proposition" involves a notion which you would seem to believe - if your assertions were credited as seriously offered - to be impossible.

    Thus,
    "By a 'proposition' I shall mean that which remains unchanged when a sentence is translated from one language into another, or from speech to writing or vice versa" Bertrand Russell, "On Verification", 1938

    ReplyDelete
  24. JA said...

    @DNW:

    Let me rephrase and extend what I meant. The argument is a variant of the ethical theory of emotivism first posited by the logical positivism of A.J. Ayer. If a claim is neither synthetic (tied to an empirical observation) nor analytic (tautological) than it is subjective and carries no content that is objectively verifiable; it is nothing more than an expression of emotion.

    Since the author identified "meaning" as something that can only reference a subjective emotion, rather than something objective or tautological, then it has no basis for public discourse--hence his drivel about personal meaning and existentialism."


    Therefore, your characterization was as inference regarding what his view would eventually entail for the propositional meaning of "meanings", under the doctrines of logical positivism.



    " Again, no serious person really argues this anymore after Quine, Wittgenstein, and Popper. Alasdair MacIntyre, an analytical Thomist, does an excellent job of both refuting emotivism and situating it as a moral pathology that developed due to historical circumstance in "After Virtue." "


    I agree that the strict Logical Positivism doctrine is pretty much abandoned by everyone other than Internet Trolls [I am not referring to anyone here with this remark] looking to pick a fight with naive Christians.

    Even Ayer, drew back from it to the extent of acknowledging that statements concerning the fact that so and so held such and such a value statement to be true, were not under his own principle of evaluating meaningful propositions, meaningless statements.



    " But something else also needs to be acknowledged: Emotivism assumes the fact-value distinction, which Aristotelians reject outright. What is going on, is that Ray is talking about meaning with different ontological and epistemological assumptions operating in the background of his thought than you are. Any dialogue with him needs to take this into account."


    You are correct that we are in part talking past one another.

    Although I don't claim to be an Aristotelian much less a Thomist, it seems to me and always has, that the kind of relativism that these sorts espouse is self-refuting, and no liar's paradox-like solution is going to save them from that fact when they start speaking in categorical terms.

    And I tend to think that talk of "natures" makes enough sense that values statements can be persuasively derived from propositions describing these natures.

    Certainly this view has a history long antedating Aristotle, and can be traced out in the etymology of the very terms that we use for right and wrong.

    ReplyDelete
  25. Grodrigues:

    First, my proposal is not meant to exaust all possibilities; the point to stress is probably more the fact that science, in the wider sense, is not reduced to the mathematization of observed correlations. The main aim of science is understand, to make intelligible. We should remind ourselves that from the level of biology upwards, mathematics has little to no more than a nominal presence. By the way, have you followed the link to the Telic Thoughts blog that someone provided some posts back? Some good stuff there.

    When you say that there are entities and phenomena that cannot be represented by a mathematical variable, what exactly do you mean? Do you mean that it cannot be done at all? Do you mean that it cannot be done with adequate precision? I only ask, because one can turn anything into a variable, but whether that can be done with any accuracy is another matter. For example, there are rating scales for depression, which assign a variable to an emotion. Certainly, this is imprecise, but is it impossible?

    Also,

    You would have a point if rationality is reducible to mathematics. As I said above, from biology upwards, there is very little mathematization going on but there is still an impressive shared fund of accumulated knowledge.

    Not necessarily. Mathematics and rationality would only have to share the property of only capturing a subset of reality in order for the analogy to hold. I think that we agree that mathematics is limited in terms of fully capturing all of reality, and we disagree regarding rationality. Other than as a guiding assumption, why believe that all of reality is understandable by reason? Where does reason get this power and capacity?

    It is not and cannot be a "guiding assumption", because it is the very presupposition for knowing the world in the first place! Your musings puzzle me because my intuitions lead me the opposite way. Although I have no illusions about human limitations, the fact is that I cannot even imagine what an uncaused event would look like. Is it contingent? then it must have a cause. To quote from the great mathematician D. Hilbert: "Wir mussen wissen. Wir werden wissen." or "We must know. We shall know."

    I am sure you cannot imagine what a sound looks like, but there are people who experience such things regularity. And it seems that you are operating between the two poles of (a) knowing that there are limitations to our capacities, and (b) not knowing how to identify those limitations. These two ideas are in constant tension, much like faith and certainty, and much of the impetus of knowledge comes from that tension. In other words, pushing our limits is what the search for knowledge is all about. As such, I think that we share a common position.

    ReplyDelete
  26. Brian:

    I think dguller is confusing the concept of nothing, which is something, with nothing. Nothing "is" just nothing.

    I am not talking about the concept, but about the content of the concept. Yes, the “concept” of X is something, but I am talking about what the concept is trying to capture, i.e. “X”. When it comes to non-Being, this is a concept that we can only acquire through the negation of Being, and thus within the content of the concept is Being being negated, and thus the content contains Being, meaning it is not purely non-Being, but only the negation of Being.

    ReplyDelete
  27. Josh:

    You can't. Let me try this again. In any judgment, any proposition you can come up with, both terms are always implicit due to the principle of non-contradiction. You can't get around this if you want to reason.

    Right.

    Implied in this judgment is the acceptance of X or not-X. And you understand completely what either X or not-X means. Appeals to agnosticism won't help, because every meaningful proposition has a truth value by nature, whether we know the answer or not.

    Right, but I am not talking about any not-X, but only about non-Being. You are absolutely right that any proposition p implies that either p or not-p is true, and thus inherently involves both an affirmation and a negation. And I agree with you that this is not a problem for the vast majority of propositions.

    Either I am sitting at the computer or I am not sitting at the computer. Yes, the latter implicitly involves the former, but it is still comprehensible, because even if I am not at the computer, I can comprehend an empty chair and desk. In other words, the negation of my being at the computer is still something. So, you can still get something from something. Things become tricky when the negation of something is nothing, because all negations involve something.

    I hope that helps.

    ReplyDelete
  28. "I am sure you cannot imagine what a sound looks like, but there are people who experience such things regularity."

    Synesthesia is not a case of people "seeing sound" technically. It's a case of "automatic, involuntary experiences in a second sensory or cognitive pathway." Correlation != causation.

    "Other than as a guiding assumption, why believe that all of reality is understandable by reason?"

    Is it more reasonable to regard reality as comprehensible or not-comprehensible?

    ReplyDelete
  29. dguller,

    Consider this selection from Wilhelmsen's Man's Knowledge of Reality: An Introduction to Thomistic Epistemology:

    "There is only one absolute Nothing...the nonbeing of the true negative judgment is not an absolute nothing: nonbeing...exercises an intelligible act of existing, an act of being-true, in every true negative judgment, even to the extreme of all negations: 'nonbeing is nonbeing.' The only pure Nothing is the Nothingness of the simultaneous affirmation that being is and is not:

    'This simultaneous affirmation and negation can have no relation at all to the intelligibility of being, not even that of non-being; for its being destroys its non-being, and its non-being destroys its being.' (St. Thomas, De Potentia Dei, Q. 1, a. 3, c.)

    To assert being both to be and not to be is to say nothing whatever. To deny the principle of contradiction is to put forth a desire; it is not to intend a reality." (pp. 152-3), emphasis added

    You can have your pure Nothing, unsoiled with Being, only at the cost of the denial of the principle of contradiction. To deny the intelligibility of nonbeing as a product of the intellect is to deny the ability to make a true judgment about anything, as noted above. The paragraphs before in Wilhelmsen's book describe the process of how we conceive nonbeing in Scholastic technical terms, but it's tangential and would make this post too long.

    ReplyDelete
  30. @dguller:

    "When you say that there are entities and phenomena that cannot be represented by a mathematical variable, what exactly do you mean? Do you mean that it cannot be done at all? Do you mean that it cannot be done with adequate precision? I only ask, because one can turn anything into a variable, but whether that can be done with any accuracy is another matter. For example, there are rating scales for depression, which assign a variable to an emotion. Certainly, this is imprecise, but is it impossible?"

    As it regards my proposal 3.? It cannot be done, at least if we are to escape Bell's theorem. But your question is not a well-posed one because precise or imprecise, as you seem to be using the terms, are not attributes of the theory itself, but attributes of the measurements.

    "Other than as a guiding assumption, why believe that all of reality is understandable by reason? Where does reason get this power and capacity?"

    Because God is rational (in the analogical sense) and the created order reflects His glory, thus it is rational. I suppose this argument does not convince you, not even if I add some scriptural quotations, so take it as a "guiding assumption" since without it, well the discussion is over. First principles are not demonstrable, only justifiable. In that direction, why believe that reality is not rationally understandable? Do you know of any phenomena that is so intractable as to be declared beyond reason? Note that the crucial split here is not between knowing everything and knowing something, not even between knowing something and knowing nothing, but between the knowable and the un-knowable. Maybe failing to make this distinction is why you keep circling around this matter? The limitations I talk about refer to the first divide.

    ReplyDelete
  31. DNW - "And certainly you do not deny that propositions exist or have objectively ascertainable meanings?"

    Let's take a step back. When discussing the 'meaning of life', are we talking about propositions?

    I mean, if your spouse says to you, "You mean so much to me", are they in effect saying, "Your existence expresses a proposition to me"? (If so, what is that proposition? Can you translate it to English?)

    ReplyDelete
  32. Mr. Green:

    But we don't need to postulate uncaused events! We have (Aristotelian/Scholastic) philosophies that explain reality without doing so, so one must be pretty desperate to refuse them and pick a "system" that's unreasonable instead.

    Ultimately, there is no argument to be made about this issue, because it is about differing intuitions. My intuition is that beyond our understanding, we simply do not know what is happening. In that case, either what is happening is similar to what we know, or is different from what we know. The bottom line is that until it is subsumed under our understanding, we just don’t know. That means that we should all just take it easy, and admit that we have no idea what we are talking about.

    They depend upon reason. Intuition may give us some ideas to get going, but in the end the argument doesn't depend on it, or it isn't a proper metaphysical argument at all. That is, metaphysics will say things like "If something comes into being, it must be caused." Now, we need some experience to apply this proposition. ("X came into being. Therefore X was caused.") However, the conditional itself does not depend on our experience of any X. In that sense, metaphysics does not depend on our questionable experiences. The principle remains even if there were no Xs anywhere.

    And where does reason get its principles? Do they automatically turn on as soon as reason is mature? Where do those principles come from? How do we know that they are reliable?

    My understanding is that even Aristotle recognized that universal principles require the experience of particular events, and abstracting the general forms of those particular events. It is not as if reason already contained all these general forms a la Plato’s Meno, but rather that its interaction with particular events in the world involves a transaction of sorts.

    Anonymous:

    Synesthesia is not a case of people "seeing sound" technically. It's a case of "automatic, involuntary experiences in a second sensory or cognitive pathway." Correlation != causation.

    Actually, that is exactly what it is. Sure, it is due to some abnormalities in their neurobiological pathways that process sensory information, but their sensory modalities are still mixed up. My point was that what we consider possible based upon our experiences is not necessarily the sum totality of what is possible, and that there are examples, such as synesthesia, where the impossible happens. That should cause us to, at least, pause when we conduct metaphysical speculations.

    Is it more reasonable to regard reality as comprehensible or not-comprehensible?

    Here’s the thing. I am not taking a position either way. All I know for certain is that some of reality is comprehensible. As for the rest, I don’t know. We will continue to push to understand as much as we can, and whatever we understand is comprehensible for sure. However, we really cannot know for certain whether the rest is comprehensible or not, and so should just try to comprehend it based on past successes, while being mindful that our limit might be just around the corner. That’s all. I am not making an absolute truth claim, but am resolutely refusing to do so, because of my own epistemic limitations. I honestly do not know why it is so objectionable to be humble in the face of the unknown.

    ReplyDelete
  33. Ray Ingles said...

    DNW - "And certainly you do not deny that propositions exist or have objectively ascertainable meanings?"

    Let's take a step back...."


    Yes, let's.

    Did you look up the definition of the word "objective" yet?


    Are you actually asserting that the rules governing the sensible use of the words "meaning" or "point", entail that they be employed as strict synonyms only for the term "felt emotion"?


    And way back to the beginning of this exchange: Are you denying that it makes linguistic sense to deny that the universe or the elements in it have a point, or, a purpose?

    You can answer as you please, or not. I say "or not", because as you have no doubt noticed by now, the predicate of this line of exchanges - whether it is legitimate to say that the universe or the things in it are pointless", or whether it is fair to characterizes others as saying it - has been obsoleted by Feser's most recent posting on Rosenberg.

    Rosenberg is after all standing there and waving the very flag you were suggesting didn't exist when I pointed to it.

    Maybe you should write to Rosenberg ...

    ReplyDelete
  34. Josh:

    You can have your pure Nothing, unsoiled with Being, only at the cost of the denial of the principle of contradiction. To deny the intelligibility of nonbeing as a product of the intellect is to deny the ability to make a true judgment about anything, as noted above. The paragraphs before in Wilhelmsen's book describe the process of how we conceive nonbeing in Scholastic technical terms, but it's tangential and would make this post too long.

    Thanks for the time you have put into looking into this matter.

    It seems that what your quote is saying is two things.

    First, that affirming a contradiction is “the only pure Nothing”. And to that I would side with Wittgenstein that a contradiction does not refer to anything at all. It is a combination of terms that results in nonsense, much like a word salad. It only superficially seems to possess sense by virtue of its surface structure, but ultimately is incoherent.

    Second, that negating the positive content of a proposition “is not an absolute nothing”. In other words, when you negate a proposition p, then you do not have pure non-Being as a result, but rather a different kind of being, i.e. one in the absence of the situation described by p. That seems similar to my point in an earlier comment that this is an example of negating something to get something else, which I have no problem with. The difficulty is when you try to negate something to get nothing, and that is where things get tricky, because all negation necessarily contains the affirmation, and the negation of Being necessarily contains Being, and thus cannot be pure non-Being in the same way that 0.00000000001 is not the same as 0.

    I think that all talk about non-Being occurs at the very limit of intelligibility, and thus involves the tension of alternating between nonsense and sense that negates the intended meaning, and thus never actually results in what one intends to refer to. It is like talking about God: he has the property X (by metaphysical deduction), but really he doesn’t have X (by virtue of our use of analogy), except that he does have X (by metaphysical deduction), except that he doesn’t have X (by virtue of our use of analogy), and so on. It is like a snake eating its own tail, and you never really get anywhere.

    Any thoughts?

    ReplyDelete
  35. E.H. Munro,

    "Evolution occurs whether or no anyone observes it, "sexiness" only occurs in the mind of the observer."

    Not hardly. Sexiness is a huge part of evolution.

    ReplyDelete
  36. Grodrigues:

    As it regards my proposal 3.? It cannot be done, at least if we are to escape Bell's theorem. But your question is not a well-posed one because precise or imprecise, as you seem to be using the terms, are not attributes of the theory itself, but attributes of the measurements.

    The question is why it cannot be done. Why can’t those properties be represented as mathematical variables? Or is it that they cannot be represented by the specific mathematical variables that QM utilizes? Or maybe for some other reason that I’m missing?

    Because God is rational (in the analogical sense) and the created order reflects His glory, thus it is rational. I suppose this argument does not convince you, not even if I add some scriptural quotations, so take it as a "guiding assumption" since without it, well the discussion is over. First principles are not demonstrable, only justifiable. In that direction, why believe that reality is not rationally understandable? Do you know of any phenomena that is so intractable as to be declared beyond reason? Note that the crucial split here is not between knowing everything and knowing something, not even between knowing something and knowing nothing, but between the knowable and the un-knowable. Maybe failing to make this distinction is why you keep circling around this matter? The limitations I talk about refer to the first divide.

    First, what does it mean to be “rational”?

    Second, I start with the fact that we know many things. I presume that you start in the same place as I do. Given that starting point, how does not infer that because we know many things that we must be able to know everything? That is the deduction that you are trying to make. And if you are not making such a deduction, then you are relying upon first principles, i.e. everything is knowable and comprehensible. That seems grandiose to me, because it makes our reason theoretically unlimited. My intuition is that we are finite beings with limitations, and that those limitations possibly transfer to our rational faculty’s capacity to uncover all truth in the universe. However, I do not make a position either way, and only say that I do not know. Only time will tell if the assumption is actually true or false. I really do not know why this is so objectionable. Do you really believe that all aspects of reality are theoretically comprehensible to the human mind? Why would you believe this?

    ReplyDelete
  37. DNW - "Are you actually asserting that the rules governing the sensible use of the words "meaning" or "point", entail that they be employed as strict synonyms only for the term "felt emotion"?"

    No, I am not. However, when speaking of finding 'meaning in or point to life', I have great difficulty understanding those terms in any other way. Care to answer my question about what your spouse means when they say, "You mean so much to me?"

    "Are you denying that it makes linguistic sense to deny that the universe or the elements in it have a point, or, a purpose?"

    Depends on the linguistic sense of the terms you're using. Are you saying that 'the universe or the elements in it don't express any propositions', or are you saying 'the universe or the elements in it cannot be of any significance or importance to anyone'? I've asked you for clarification on that point a couple times now.

    "Rosenberg is after all standing there and waving the very flag you were suggesting didn't exist when I pointed to it."

    When did I say that the flag didn't exist?

    (And why can I not disagree with you and Rosenberg?)

    ReplyDelete
  38. dguller,

    Leaving aside the argument about analogy and how it applies, let's take a look back:

    First, that affirming a contradiction is “the only pure Nothing”. And to that I would side with Wittgenstein that a contradiction does not refer to anything at all. It is a combination of terms that results in nonsense, much like a word salad.

    This is restating what Wilhelmsen is saying; "pure Nothingness" is the simultaneous affirmation and denial of Being. So then we agree: a pure, absolute Nothing is incoherent in principle.

    Second, that negating the positive content of a proposition “is not an absolute nothing”. In other words, when you negate a proposition p, then you do not have pure non-Being as a result, but rather a different kind of being, i.e. one in the absence of the situation described by p. That seems similar to my point in an earlier comment that this is an example of negating something to get something else, which I have no problem with. The difficulty is when you try to negate something to get nothing, and that is where things get tricky

    Here we also agree, but I think you are confused on this last part. Wilhelmsen affirms the ideal existence of the concept of nonbeing, even in the judgment, "nonbeing is nonbeing." So when we "negate something to get nothing," as you say, it is just like anything else actually; it is "negating something to get something else," in this case, the ideal concept of 'nonbeing' formed by a reflective action of intellect.

    That is why we can meaningfully refer to 'nothing' and 'nonbeing' as objects of thought. I believe the reason why you have trouble is seen in your notion above that when we are talking about 'nothing,' we are "trying to get something" from the "absolute nothing" referred to above. But we both agree that's impossible.

    We are able to form the concept of nonbeing from being, like how "shadows only live when given birth by light."

    ReplyDelete
  39. Josh:

    Here we also agree, but I think you are confused on this last part. Wilhelmsen affirms the ideal existence of the concept of nonbeing, even in the judgment, "nonbeing is nonbeing." So when we "negate something to get nothing," as you say, it is just like anything else actually; it is "negating something to get something else," in this case, the ideal concept of 'nonbeing' formed by a reflective action of intellect.

    You seem to agree that non-Being is “something else”, but that would mean that non-Being is a different manifestation of Being, which it necessarily cannot be. In other words, it is what it cannot be, and thus is incoherent, which is exactly my point. Again, non-Being is different from different manifestations of Being, which is why you can use the law of non-contradiction when discussing different manifestations of Being, but it falls into incoherence when you try to talk about non-Being, because it inevitably turns non-Being into a type of Being, which is nonsense.

    That is why we can meaningfully refer to 'nothing' and 'nonbeing' as objects of thought. I believe the reason why you have trouble is seen in your notion above that when we are talking about 'nothing,' we are "trying to get something" from the "absolute nothing" referred to above. But we both agree that's impossible.

    It is not a notion, but a necessity. You agree that the law of non-contradiction is coherently applicable when discussing different manifestations of Being. You also agree that whenever we have the concept “not-X” that it inevitably involves “X”. And if that is true, then our concept of non-Being inevitably involves Being, and thus cannot be non-Being at all, but rather a different type of Being, which is nonsensical. And that is why I think that the concept of non-Being is incoherent, it stretches the limit of thought into nonsense, and thus is effectively a string of words that means nothing at all. And if that is true, then talking about what follows from nothing, i.e. from nothing, nothing comes, is equally incoherent.

    Any thoughts?

    ReplyDelete
  40. @dguller:

    "The question is why it cannot be done. Why can’t those properties be represented as mathematical variables? Or is it that they cannot be represented by the specific mathematical variables that QM utilizes? Or maybe for some other reason that I’m missing?"

    Are you going loopy? *You* yourself answered the question by dragging in Bell's theorem. If it is to be a *local* variable (variable meaning "distribution on a probability space with a well-defined probability and expectation value") then the violation of Bell inequalities observed in nature rules it out.

    "First, what does it mean to be “rational”?"

    Skepticism is addictive, isn't it? Start by asserting that reason is powerless in some domains, conveniently left unspecified, and end up asking what it means to be "rational". Until now, we both have been using the word without any troubles in communication, so why this change of heart? If I offer a definition will you then ask the meaning of some word in the definition, and so on and on ad infinitum?

    "My intuition is that we are finite beings with limitations, and that those limitations possibly transfer to our rational faculty’s capacity to uncover all truth in the universe. However, I do not make a position either way, and only say that I do not know. Only time will tell if the assumption is actually true or false. I really do not know why this is so objectionable. Do you really believe that all aspects of reality are theoretically comprehensible to the human mind? Why would you believe this?"

    Have you not read my post? I told you why I believe in it (in fact, I already mentioned this in a previous post when I spoke about the fundamental differences in worldview of a classical theist and an atheist). I followed by saying that the argument is bound not to convince you, so take it as a first principle and work up to justify it. I was even explicit about the fact that we may be unable to "uncover all truth in the universe" but that that is not relevant for my contention. And contrary to what you say, *because* we are finite beings the question *may not* be decided with time.

    1. Do you have any example of some phenomena that is so intractable that it is declared un-knowable? The fact that there isn't shows the evidence is overwhelmingly on my side. As an inductive argument, I reckon that it is in pretty good shape.

    2. How would you recognize un-knowable phenomena? I cannot imagine it, but maybe you have some clear grasp of it you would like to share with us?

    3. To borrow from Verbose Stoic's above post, to know X is already to presuppose that X is knowable. In order to see if our presupposition is right, we try to delineate more precisely the concept of X. But in doing so, we automatically gain some insight into X, such as that X is this or that X is not that. Further investigations may or may not point to fundamental limitations in our knowledge of X, but the fact that these investigations can be conducted in the first place is proof that the concept we entertain of X is in part known, and thus knowable.

    This has already been dragging for too long, so until you meet the above objections, I think I am rationally entitled to my position and need not say anything more in its defence. You want to string together some mights and some ifs and proclaim that maybe, just maybe, reason is *fundamentally* powerless in some domain, that for all we know may not even exist, and then congratulate yourself on how humble and reasonable (heh) you are? Go ahead, be my guest.

    ReplyDelete
  41. Grodrigues:

    Are you going loopy? *You* yourself answered the question by dragging in Bell's theorem. If it is to be a *local* variable (variable meaning "distribution on a probability space with a well-defined probability and expectation value") then the violation of Bell inequalities observed in nature rules it out.

    Let’s recall what the point of this discussion has been. I am open to the possibility of uncaused events, and I cited Bell’s inequality as opening up the possibility of variables that would be able to explain the random quantum phenomena observed in experiments, but which are hidden from our view, whether due to ontological or epistemological factors. And since they are hidden, we really cannot comment on them, and thus cannot say for certain that they are identical to other causes that we can identify. They might, but then again, they might not. Since they are hidden, who knows?

    You then cleverly replied that there may be a natural cause that underlies such phenomena in a way that preserves causation, but that it simply cannot be captured by any mathematical formalism, and thus cannot be a variable at all in any equation, which would not make it a hidden variable by virtue of not being a variable at all. I then asked why it cannot be a variable, and you seem to be replying that if it were, then Bell’s inequality would apply, and then it would count as a variable after all, and thus be a hidden one, making us unable to comment upon it, which would demonstrate my position. In other words, your position is based upon the fact that it must be the case, or else I am correct.

    I’m sure I’m missing something here, because that doesn’t seem compelling to me.

    Skepticism is addictive, isn't it? Start by asserting that reason is powerless in some domains, conveniently left unspecified, and end up asking what it means to be "rational". Until now, we both have been using the word without any troubles in communication, so why this change of heart? If I offer a definition will you then ask the meaning of some word in the definition, and so on and on ad infinitum?

    Offer a definition, and we’ll take it from there. I’ve noticed that in Thomism, ordinary terms are used with specialized meaning, and I’m curious about your specific meaning, because it might be a technical one that I am missing. There is no point arguing when the terms aren’t clear. In fact, it would be considered arguing against a straw man, which I usually try to avoid when I can.

    1. Do you have any example of some phenomena that is so intractable that it is declared un-knowable? The fact that there isn't shows the evidence is overwhelmingly on my side. As an inductive argument, I reckon that it is in pretty good shape.

    Then it’s not an assumption at all. It is a conclusion based upon induction, and induction is never 100% certain.

    ReplyDelete
  42. grodrigues:

    2. How would you recognize un-knowable phenomena? I cannot imagine it, but maybe you have some clear grasp of it you would like to share with us?

    This is why I find these issues to fascinating. By virtue of our finite position and facing an unknown future, making universal statements should always be hedged with some degree of uncertainty. So, even if we confronted a phenomena that is simply incapable of being understood by all our logical, rational and empirical tools – perhaps because they always lead to contradiction and incoherence despite the fact that the phenomena is staring us in the face – then it is always possible that the next thinker will find a way to understand it.

    Perhaps a good example from Christianity would be the Incarnation and even the Trinity. No matter how many different ways one approaches them, there seems to always be a point where they are declared a Mystery, beyond human understanding.

    to know X is already to presuppose that X is knowable. In order to see if our presupposition is right, we try to delineate more precisely the concept of X. But in doing so, we automatically gain some insight into X, such as that X is this or that X is not that. Further investigations may or may not point to fundamental limitations in our knowledge of X, but the fact that these investigations can be conducted in the first place is proof that the concept we entertain of X is in part known, and thus knowable.

    Not necessarily. Say you have X, and you try to understand it one way, but it leads to a contradiction. Then you try to approach it another way, and it leads to a contradiction. Then you try a third approach, and it leads to another contradiction. Would you really say that you have gained some insight into X when all attempts have resulted in incoherence?

    This has already been dragging for too long, so until you meet the above objections, I think I am rationally entitled to my position and need not say anything more in its defence. You want to string together some mights and some ifs and proclaim that maybe, just maybe, reason is *fundamentally* powerless in some domain, that for all we know may not even exist, and then congratulate yourself on how humble and reasonable (heh) you are? Go ahead, be my guest.

    Thanks. I think I just might do that. Ahhhh. That pat on the back feels really nice. ;)

    ReplyDelete
  43. Ray Ingles said...

    DNW - 'Are you actually asserting that the rules governing the sensible use of the words "meaning" or "point", entail that they be employed as strict synonyms only for the term "felt emotion"?'

    No, I am not."

    Ok then, one objection disposed of. It is agreed that it is not a violation of a linguistic rule to say that (some people believe that) patterns or life have no point; on the supposed basis that to say "have a point", must be restricted to a reference to an emotional state.

    " However, when speaking of finding 'meaning in or point to life', I have great difficulty understanding those terms in any other way."


    Fine. As long as you recognize that you are representing some mental quirk of your own, and not advancing a point of logic or some indubitable metaphysical principle.


    " Care to answer my question about what your spouse means when they say, "You mean so much to me?" "


    No, I would not care to since she is irrelevant to the matter I was discussing. I was not referring to subjective feelings, and I don't find the discussion to be of much philosophical or logical relevance here. I have however slogged through Sein und Zeit in translation, and if you want to try and relate moods to the explication of existence go ahead. But ... don't expect me to follow.



    " 'Are you denying that it makes linguistic sense to deny that the universe or the elements in it have a point, or, a purpose?'

    Depends on the linguistic sense of the terms you're using. Are you saying that 'the universe or the elements in it don't express any propositions', or are you saying 'the universe or the elements in it cannot be of any significance or importance to anyone'? I've asked you for clarification on that point a couple times now."


    Geez .... In the passage you originally objected to, I was characterizing the view Rosenberg was later - thanks to Feser - seen to express almost as soon as I characterized it. You challenged my characterization by attacking - by implication via your link - the notion that such a view was intellectually coherent enough to even be assigned a truth value.



    " "Rosenberg is after all standing there and waving the very flag you were suggesting didn't exist when I pointed to it."

    When did I say that the flag didn't exist?"


    Your little buddy denied that the figurative text on our metaphorical flag had an intellectually coherent meaning because, "meaning" under his rule of interpretation could only refer to the subjective experience of an emotion, or to a subjective experience and an emotion ... or to whatever it was he was trying to say ...


    " (And why can I not disagree with you and Rosenberg?)"

    You can disagree with Rosenberg, but you cannot deny that I accurately if flippantly characterized the view of people like him. Nor, now since your admission at the top, can you disagree that the proposition Rosenberg advances regarding the objective meaninglessness of life, has linguistic meaning, whether it is true or false.

    That then, resolves down to you disagreeing with Rosenberg on the cosmic (if we may use that word for the sake of convenience) significance of your own subjective feelings of having a point.

    That's a wrap, as they say, eh?

    ReplyDelete
  44. DNW - "Geez .... In the passage you originally objected to, I was characterizing the view Rosenberg was later - thanks to Feser - seen to express almost as soon as I characterized it."

    Ah, but you presented it as 'the atheist position': "If to live rightly is to live as an atheist, and to live as an atheist is to live without illusions, let us do so."

    It'd be like me saying "Christians believe justification is by faith and works." It is, at best, woefully incomplete.

    "Your little buddy denied that the figurative text on our metaphorical flag had an intellectually coherent meaning..."

    Er... no he didn't, as I pointed out several times. He was speaking of the experience of meaning or significance, and he was pretty explicit about that.

    Of course, now you're claiming not to be talking about 'subjective feelings'... so in your original 'flippant characterization' you must have meant 'the universe or the elements in it don't express any propositions'. It's not clear to me how the universe could express a proposition. But I'd say an atheist could certainly believe that elements in the universe like, say, people, could express propositions.

    ReplyDelete
  45. @dguller:

    "You then cleverly replied that there may be a natural cause that underlies such phenomena in a way that preserves causation, but that it simply cannot be captured by any mathematical formalism, and thus cannot be a variable at all in any equation, which would not make it a hidden variable by virtue of not being a variable at all. I then asked why it cannot be a variable, and you seem to be replying that if it were, then Bell’s inequality would apply, and then it would count as a variable after all, and thus be a hidden one, making us unable to comment upon it, which would demonstrate my position. In other words, your position is based upon the fact that it must be the case, or else I am correct."

    Here is the recap from my point of view. As far as my reply goes, if you want to retain locality, and assuming all other loopholes I mentioned are closed, then the natural cause cannot be a variable in the sense of a distribution on a probability space with a well defined probability and expectation value, because the Bell inequalities rule it out. Way out: drop the variable requirement. Understanding, including causality, is not reduced to what is amenable to a mathematical treatment as I have been repeating ad nauseam. If you want to make *quantitative* predictions, then the variable requirement is pretty much a given. If you drop this requirement, then there *may* be mathematical formalisms that could make *qualitative* predictions -- think of classical systems exhibiting chaotic behavior where only qualitative features can be described. It seems improbable to me but I have no argument to rule it out.

    "Offer a definition, and we’ll take it from there. I’ve noticed that in Thomism, ordinary terms are used with specialized meaning, and I’m curious about your specific meaning, because it might be a technical one that I am missing. There is no point arguing when the terms aren’t clear. In fact, it would be considered arguing against a straw man, which I usually try to avoid when I can."

    I am using the word "rational" in its common ordinary meaning. The online dictionary lists: Having or exercising the ability to reason; consistent with or based on reason; logical.

    "Do you have any example of some phenomena that is so intractable that it is declared un-knowable? The fact that there isn't shows the evidence is overwhelmingly on my side. As an inductive argument, I reckon that it is in pretty good shape.

    Then it’s not an assumption at all. It is a conclusion based upon induction, and induction is never 100% certain."

    Huh... I did say it was an inductive argument to justify, not prove, a first principle. What more do you want?

    ReplyDelete
  46. @dguller (continued):

    "Perhaps a good example from Christianity would be the Incarnation and even the Trinity. No matter how many different ways one approaches them, there seems to always be a point where they are declared a Mystery, beyond human understanding."

    Those are indeed good examples. Until now I was operating under the assumption that we were talking about the natural order only; bringing in the Divine order is changing the goal posts (although the Incarnation is the intersection of the Divine with the natural order or as T. S. Eliott puts it "the still point of the turning world") -- but arguing with a Christian against rationality by citing the Christian mysteries of revealed knowledge is such a delightful irony, that just for pulling this stunt you deserve another pat on the back.

    There are obvious limits to our knowledge of divine mysteries, since by definition they are part of the communication of an infinite mind to a finite mind. As a first answer I would dispute, given that they are part of revealed *knowledge*, that they are totally and completely un-knowable. Furthermore, they are mysterious insofar our lack of knowledge, not because they are inherently irrational, which they could never be because God is rational (in the analogical sense). As St. Paul says "For now we see only a reflection as in a mirror; then we shall see face to face. Now I know in part; then I shall know fully, even as I am fully known." (1 Corinthians 13:12). Saying more would lead me into the relation of revealed knowledge with reason, a subject I am woefully incompetent to say anything mildly relevant.

    "Not necessarily. Say you have X, and you try to understand it one way, but it leads to a contradiction. Then you try to approach it another way, and it leads to a contradiction. Then you try a third approach, and it leads to another contradiction. Would you really say that you have gained some insight into X when all attempts have resulted in incoherence?"

    What do you mean when you say you have X? I take it you mean that we have formed a concept in the intellect of X; but this entails that X is something definite. There are some things that it is, and somethings that it is not. If it is consistent (and what else can it be, if it is a concept of something real?) then by a via negativa we can prune all the incoherencies. What subsists cannot be the concept of non-being, because the hypothesis is that it is the concept of something existing.

    "That pat on the back feels really nice."

    Happy to oblige. Your obedient servant,
    G. Rodrigues

    ReplyDelete
  47. dguller,

    I have a hard time understanding why the concept of non-being presented in the PNC:

    “It is impossible to hold (suppose) the same thing to be and not to be (Metaph IV 3 1005b24 cf.1005b29–30).”

    Is any less usable/intelligible in this:
    Out of non-being/nothing, non-being/nothing comes

    If non-being is unintelligible as you say, then we have no right to regard the PNC as a binding principle of reason.

    Another outcome of this is that, if non-being as a concept is unintelligible, then being as a concept is unintelligible. We would have no warrant to say anything is unless we can understand how it isn't.

    Non-being is intuitively obvious to our speculative intellect, and it's only when we try to treat it as some sort of Platonic form as opposed to a purely ideal object that we get mystified.

    ReplyDelete
  48. Josh:

    If non-being is unintelligible as you say, then we have no right to regard the PNC as a binding principle of reason.

    I think that there is an ambiguity here that may be causing the confusion.

    It is coherent to talk about a specific state of affairs either occurring or not occurring, but only in the context of the rest of reality still existing. For example, it is coherent to say that either it is raining or it is not raining, because whether it is raining or not, the rest of the world still exists. In that scenario, the affirmation involves something existing (i.e. there is a world in which it is raining) and the negation also involves something existing (i.e. there is a world in which it is not raining). In that sense, non-being makes sense, but it refers to a specific state of affairs not occurring, and in that sense, the PNC nicely operates without incoherence.

    The situation seems to be different when we are not talking about non-being of a specific state of affairs, but about non-Being, i.e. the complete non-existence of anything. My argument is that such a notion is completely incoherent, because to conceive of non-Being, one must negate Being, and thus our concept of non-Being always involves Being, and thus is not non-Being at all.

    If your problem with this is that it seems to nullify the PNC, then I don’t think you have anything to worry about. The PNC still applies nicely when the affirmation and negation both still represent some actual state of affairs in which either some specific situation either exists (i.e. the affirmation) or does not exist (i.e. the negation). However, I don’t think it holds when it applies to the negation of Being, because we cannot even form a concept of such a thing.

    Another outcome of this is that, if non-being as a concept is unintelligible, then being as a concept is unintelligible. We would have no warrant to say anything is unless we can understand how it isn't.

    Not necessarily. Take the proposition that “there exists a square circle”. That proposition is incoherent, because it contains a concept that is incoherent, i.e. a square circle. However, the negation of the proposition is perfectly sensible, i.e. “there does not exist a square circle”. It thus follows that if p is incoherent, then not-p may still be coherent, and thus if non-Being is incoherent, then Being may still be coherent.

    Any thoughts?

    ReplyDelete
  49. Grodrigues:

    Here is the recap from my point of view. As far as my reply goes, if you want to retain locality, and assuming all other loopholes I mentioned are closed, then the natural cause cannot be a variable in the sense of a distribution on a probability space with a well defined probability and expectation value, because the Bell inequalities rule it out. Way out: drop the variable requirement. Understanding, including causality, is not reduced to what is amenable to a mathematical treatment as I have been repeating ad nauseam. If you want to make *quantitative* predictions, then the variable requirement is pretty much a given. If you drop this requirement, then there *may* be mathematical formalisms that could make *qualitative* predictions -- think of classical systems exhibiting chaotic behavior where only qualitative features can be described. It seems improbable to me but I have no argument to rule it out.

    Honestly, this is above my pay grade since I’m no mathematician, so I’ll trust you on this. But I have to say that your idea that the underlying causes of quantum phenomena not necessarily being quantifiable in a mathematical formalism is highly intriguing, and it is something I have not come across in all the accounts of Bell’s inequality and its ontological implications. Thanks for sending it my way. Definitely something to reflect upon.

    I am using the word "rational" in its common ordinary meaning. The online dictionary lists: Having or exercising the ability to reason; consistent with or based on reason; logical.

    Thanks. Just wanted to make sure about the meaning of the term. I understand “logical” to mean the capacity to make inferences between propositions that preserve truth-values. The concept of “reason” seems broader than “logic”, though, because logic simply deals with the relationships between propositions, but not with their particular content. How does reason help to determine the truth values of the propositions themselves in a way that is independent of just the logical relationship of those propositions to other propositions?

    I suppose that is where sensory input is required to provide the content for reason to operate upon, and its capacity to identify possible general patterns within specific instances would be its paradigmatic capacity. From a Thomist standpoint, I guess that would be the intellect’s capacity to identify the forms of particular entities.

    Anyway, this would probably get us farther afield than necessary. My question was where reason gets the power to comprehend the entire universe. You answered that the universe is comprehensible by virtue of God’s rationality. My problem is that God’s rationality is supposed to be demonstrated by reason, and thus we are stuck in a circular account. Reason implies God, and God implies reason. In some ways, it is similar to Descartes’ solution to his skeptical doubts.

    ReplyDelete
  50. Grodrigues:

    Huh... I did say it was an inductive argument to justify, not prove, a first principle. What more do you want?

    You’re right. Those are different things, which I conflated erroneously. But the point still stands that you are applying a model (i.e. the comprehensibility of the universe by reason) that has held up very well until now, but the future is always an open question. And that’s why I remain agnostic about this matter.

    Those are indeed good examples. Until now I was operating under the assumption that we were talking about the natural order only; bringing in the Divine order is changing the goal posts (although the Incarnation is the intersection of the Divine with the natural order or as T. S. Eliott puts it "the still point of the turning world") -- but arguing with a Christian against rationality by citing the Christian mysteries of revealed knowledge is such a delightful irony, that just for pulling this stunt you deserve another pat on the back.

    I thought we were talking about reality. If the Divine is part of reality, and reality is comprehensible, then the Divine must also be comprehensible. And if you are allowed to make divisions in reality between what is comprehensible and what is incomprehensible, then I think you’ve just made my point that reason’s capacity to understand reality is likely limited, which means that some aspects of reality are likely incomprehensible.

    What do you mean when you say you have X? I take it you mean that we have formed a concept in the intellect of X; but this entails that X is something definite. There are some things that it is, and somethings that it is not. If it is consistent (and what else can it be, if it is a concept of something real?) then by a via negativa we can prune all the incoherencies. What subsists cannot be the concept of non-being, because the hypothesis is that it is the concept of something existing.

    You asked for some inkling of what it might mean to have an unknowable X. What I meant was that X happens, and that all attempts to understand X result in contradiction, perhaps even with each other! In other words, X necessarily has both property P and not-P. That would mean that X is impossible, and yet X has happened. It would be the ultimate aporia in which all attempts at comprehension result in incoherence. Is this vague? Absolutely. It has to be, because nothing like X has ever happened, and hopefully never will.

    Happy to oblige. Your obedient servant,
G. Rodrigues

    "Servant"? We are all equal. But some are more equal than others. ;)

    ReplyDelete
  51. Not hardly. Sexiness is a huge part of evolution.

    Two things here, first this claim is actually irrelevant to what said. In the second you've completely missed my point, namely that you made a horrifically bad analogy. If you're going to disprove final causality you need to find a better analogy.

    ReplyDelete
  52. Ray,

    "No, I am not. However, when speaking of finding 'meaning in or point to life', I have great difficulty understanding those terms in any other way. Care to answer my question about what your spouse means when they say, "You mean so much to me?" "

    I think that they mean "You're very important to me". But that's not the same thing as meaning or point of life, since that clearly is NOT a synonym for "What's important to me" but is indeed closer to "My purpose".

    ReplyDelete
  53. @dguller:

    "Thanks. Just wanted to make sure about the meaning of the term. I understand “logical” to mean the capacity to make inferences between propositions that preserve truth-values. The concept of “reason” seems broader than “logic”, though, because logic simply deals with the relationships between propositions, but not with their particular content. How does reason help to determine the truth values of the propositions themselves in a way that is independent of just the logical relationship of those propositions to other propositions?"

    It does not. You need premises, either in the form of axioms, like logical axioms, deduction rules, etc., or in the form of propositions derived from sense data. This is what you seem to be saying in the paragraph following the quoted one, with which I agree completely.

    "Anyway, this would probably get us farther afield than necessary. My question was where reason gets the power to comprehend the entire universe. You answered that the universe is comprehensible by virtue of God’s rationality. My problem is that God’s rationality is supposed to be demonstrated by reason, and thus we are stuck in a circular account. Reason implies God, and God implies reason. In some ways, it is similar to Descartes’ solution to his skeptical doubts."

    I do not think there is anything circular in the reasoning. You admit that reason is not completely powerless; this is enough to get the metaphysical proofs going. If the proofs go through as the Thomist contends, then the existence of a rational God is established. From that, we can establish the rationality and intelligibility of the *whole* of reality.

    "I thought we were talking about reality. If the Divine is part of reality, and reality is comprehensible, then the Divine must also be comprehensible. And if you are allowed to make divisions in reality between what is comprehensible and what is incomprehensible, then I think you’ve just made my point that reason’s capacity to understand reality is likely limited, which means that some aspects of reality are likely incomprehensible."

    Well, since you harbor all sorts of suspicions about the reality of the Divine, it seemed to me that "reality" meant what we could both agree on. But this is just a minor misunderstanding, hardly relevant for anything said until now.

    About your specific comment, I did address the "un-knowability" concerns in the following paragraph. I think part of the problem is that we have different understandings of what intelligible and comprehensible means. You go from our epistemological limitations as finite minds, which I do not dispute, to the possibility of what in my view is the statement of a radical incomprehensibility and unintelligibility somewhere in reality. As you probably recall, I mentioned that "intelligible all the way down" is one of the crucial differences between a theist and an atheist.

    "You asked for some inkling of what it might mean to have an unknowable X. What I meant was that X happens, and that all attempts to understand X result in contradiction, perhaps even with each other! In other words, X necessarily has both property P and not-P. That would mean that X is impossible, and yet X has happened. It would be the ultimate aporia in which all attempts at comprehension result in incoherence. Is this vague? Absolutely. It has to be, because nothing like X has ever happened, and hopefully never will."

    If you arrived at the conclusion that X necessarily is P and not-P it would be a reductio ad absurdum that X does not exist. But X does exist by hypothesis, so the reasoning would be wrong somewhere, somehow. Either that, or reality itself is inconsistent; but then all science can be thrown overboard from the classical principle that from a falsity anything can be derived.

    ReplyDelete
  54. Verbose Stoic - "But that's not the same thing as meaning or point of life, since that clearly is NOT a synonym for "What's important to me" but is indeed closer to "My purpose"."

    And why would a self-determined teleology be false or wrong? If I say, "my purpose for my life is to make myself and my loved ones happy", how would you go about arguing against that?

    ReplyDelete
  55. I have some questions on the "debate about nothing"...

    Doesn't equating "nothing" to "non-being" leave open the possibility that "potential being" would qualify as "nothing"?

    For instance, if we describe "nothing" in terms of being, would "pure potential" (prime matter) still be "nothing"?

    Wouldn't "nothing" be best defined then as "lacking all potentiality and actuality"?

    It seems to me that defining "nothing" in terms of potentiality and actuality is the only way to logically conclude that it is impossible for something to come from nothing - since there can be no "potential something" in "nothing".

    Thoughts?

    ReplyDelete
  56. E.H. Munro,

    "Two things here, first this claim is actually irrelevant to what said."

    You said, "Evolution occurs whether or no[t] anyone observes it, 'sexiness' only occurs in the mind of the observer."

    I said, "Sexiness is a huge part of evolution." Do you deny this? If not, it is extremely relevant to what you said. If sexiness is a huge part of evolution then it occurs outside the 'mind' of the observer. It occurs whether the 'mind' observes it or not because it pre-dates 'mind' in the first place.

    "In the second you've completely missed my point, namely that you made a horrifically bad analogy. If you're going to disprove final causality you need to find a better analogy."

    And you're going to have to do better than that. Your one attempt to show it was bad analogy was a failure. Do you have any others? I think it's a great analogy until you show it to be otherwise.

    ReplyDelete
  57. Daniel,

    Check out this link:
    http://telicthoughts.com/empty-space-time-logical-being-real-being-or-really-really-nothing/

    It should differentiate between the philosophical nothing and the material "nothing"

    ReplyDelete
  58. Josh: "It should differentiate between the philosophical nothing and the material "nothing""

    Thanks for the TelicThoughts link (I'm a regular there but I didn't really pay much attention to that one.)

    Shouldn't they be the same thing though (the philosophical and the material "nothing")?

    Or, more correctly, shouldn't the philosophical "nothing" accurately describe the material "nothing"?

    ReplyDelete
  59. Daniel,

    They aren't necessarily the same thing, as Philosophy through Ontology can study Being as Being qua Being, which supersedes the material Being in its scope.

    However, if the scientific/material "nothing" bears a relation to prime matter, then it can certainly inform that whole discussion...

    ReplyDelete
  60. You said, "Evolution occurs whether or no[t] anyone observes it, 'sexiness' only occurs in the mind of the observer."

    Yes, is that so hard to comprehend?

    I said, "Sexiness is a huge part of evolution." Do you deny this?

    Again, it's not even remotely relevant to what I wrote. Not even a little bit. Not even a smidgeon. You may as well have written "God didn't make little green apples and it don't rain in Indianapolis in the summer time..."

    Find a better example.

    ReplyDelete
  61. Ray,

    "And why would a self-determined teleology be false or wrong? If I say, "my purpose for my life is to make myself and my loved ones happy", how would you go about arguing against that?"

    I'd ask you how you know that that's your purpose, and if you reply that purpose can only be what you decide it is I'd ask what makes you think that and that you don't have an objective purpose to fulfill.

    But you concede the point I was making, which is that you need to do more than simply listing things you find important to argue for a purpose. You have to at least declare that that IS a purpose, which can then be argued against independently over whether or not your family are important to you. So they aren't synonymous, and don't mean the same thing conceptually.

    ReplyDelete
  62. E.H. Munro,

    "Yes, is that so hard to comprehend?"

    I comprehend that it's factually wrong. Find a better reason or I'll stick with my analogy.

    ReplyDelete
  63. Josh: "However, if the scientific/material "nothing" bears a relation to prime matter, then it can certainly inform that whole discussion..."

    Let me run this through in my head...

    If the material "nothing" is prime matter, then it is conceivable for something to come from nothing since pure potential can be actualized by something actual.

    Of course, this requires that something actual already exists to actualize prime matter.

    So, it is still impossible for something to come from nothing unless something actual exists to actualize it.

    Hence "in the beginning God".

    Got it! (I think.)

    ReplyDelete
  64. Verbose Stoic - "I'd ask what makes you think that and that you don't have an objective purpose to fulfill."

    What could an 'objective purpose' possibly look like? Purposes are things that agents have for things. And different agents will have different purposes for things. Usually multiple purposes.

    The White House is where the President of the United States lives when in office. Is that its 'telos'? But wait - it's also an office building; the center of the executive branch. Is that its telos? And, hey, wait a minute - I've taken a tour of the public areas. Is the telos of the White House to be a museum? And then there's the fact that it was designed and architected specifically to impress visiting foreign dignitaries - is that its telos?

    What about natural resources? What is the 'telos' of sand? To be made into static art, like a sand painting? To be made into a dynamic tool, like an hourglass? To be made into sandbags to hold back a flood? To be squirted onto rails to improve a railroad car's traction? To be used as ballast for a balloon? To be purified to make a silicon chip? If for a silicon chip, what kind - in a Wii or an Xbox or a Playstation?

    A statue in a city square fulfills a purpose - decoration, memorial, whatever - but not only that purpose. It can serve as a landmark for navigating about the city. It can serve as a jungle gym for kids to climb on. It can serve as a perch for birds. It can serve as a blind from behind which to spy on someone. It can serve a fleeing pickpocket as an obstacle to slow pursuit. It can serve as a source of metal to melt down into cannons to defend the city. The number of different 'final causes' served by anything in the real world is probably at least equal to the number of agents that interact with it.

    Even the artisan who made the statue may have multiple purposes in constructing it. He may wish to commemorate a fallen soldier... while at the same time subtly castigating the generals who ordered the march the soldier died in. The artisan may also choose a particular artistic style in order to make a statement to some of his fellow artisans, and have chosen brass as the medium to help out his brother-in-law the metal merchant. Plus, the artisan no doubt intends to be be paid for the statue... to help keep his children fed.

    Which of these, if any, is 'the' 'objective' purpose of the statue? Why?

    ReplyDelete
  65. Ray,

    "Purposes are things that agents have for things. And different agents will have different purposes for things. Usually multiple purposes."

    And that's your whole problem: the purpose we're talking about here is the purpose it was created for. That agents may use something for other purposes never changes that.

    ReplyDelete
  66. "And that's your whole problem: the purpose we're talking about here is the purpose it was created for. That agents may use something for other purposes never changes that."

    Two questions. First, a repeat: Why only one purpose?

    Second - I'm sure that's an interesting question, but why is that purpose (assuming arguendo it was only created for one single solitary purpose) the 'objective' purpose?

    ReplyDelete
  67. I comprehend that it's factually wrong. Find a better reason or I'll stick with my analogy.

    And you also "comprehend" that mathematics are empirical. And in both cases you're wrong. You need to work on your comprehension. Sexual drive and "sexiness" are not one and the same. Your analogy remains disastrously bad.

    ReplyDelete
  68. E.H. Munro,

    "And you also "comprehend" that mathematics are empirical."

    Speaking of bad analogies. What does the math issue have to do with the sexiness issue? Besides, I don't claim mathematics are empirical. I claim mathematics is based on empirically observed first principles, like notions of equality and sets of things (numbers). And I claim that without occasional empirical feedback math truths are in doubt.

    "Sexual drive and 'sexiness' are not one and the same."

    Really? How are they different? And how is the in-your-mindness of sexiness different than the in-your-mindness of intentionality?

    ReplyDelete
  69. EH Munro: "Sexual drive and "sexiness" are not one and the same. Your analogy remains disastrously bad."

    True. Libido and sexual attractiveness are two different things. Think appetite and appetizing.

    ReplyDelete
  70. Anonymous,

    Like intentionality and intentions. E.H. Munro avoids the issue through hair-splitting.

    ReplyDelete
  71. "The terminology of 'intentionality' can also be confusing, for at least two reasons. First, intentionality has nothing in particular to do with intending, or intentions. Intentions, for instance the intention to buy a cat, are just one of many types of intentional mental states."

    http://mit.edu/abyrne/www/intentionality.html

    ReplyDelete
  72. if Searle is right, intrinsic meaning or intentionality, and thus the
    first-person point of view of the conscious, thinking subject are
    inextricably bound up together; and if the argument in the proceeding section are right, meaning or intentionality, and thus the first person point of view of the conscious etc...Dualism would seem to be vindicated...


    Yet Searle has never said he was a theist..or even a dualist IIRC. Mind could exist in some fashion --at least human thinking might be unique (and it is)---that does not necessarily imply monotheism or a "ghost in the machine"--tho many naive (or manipulative) theists insist it does.

    ReplyDelete
  73. "imply monotheism or a "ghost in the machine"--tho many naive (or manipulative) theists insist it does. "

    Strawmen.

    ReplyDelete
  74. DNW,

    The link: "It’s an experience, an emotion, not an assertion of fact."

    You: "I don't know anyone serious who would take such stuff at face value. You don't really, do you?"

    I take it seriously. It's obviously true, at least in part. Meaning is a subjective experience. It's often an emotional attachment to something like, for example, a favorite song from our youth.

    ReplyDelete
  75. Thinking “What caused God?” is a good objection to the cosmological argument is one

    Hume in his Dialogues of Natural Religion brings up something like this, however vull-gar papists find it--"Philo" asks well how do we know whether a creator was one, a thousand--Brahma, JHVH or Zeus, , whether finite or infinite, etc? (Kant also hints at the issue--at least somewhat related to actual empirical physico-astronomical issues..not the mind of Zeus)) Not real subtle for the wannabe-scholastic but...there is a matter of observation (and Feser has a great skill in making Aquinas sound like he knew what happened, personally tho AQ's off by a few billion years, and pre-Copernican--in other wrods, Feser presents ancient dogma as if it had something to do with science, and his acoyltes are bamboozled. The most he can say is something like what Searle suggests--"intention" seems to suggest Mind, and...yada yada yada)

    ReplyDelete
  76. "imply monotheism or a "ghost in the machine"--tho many naive (or manipulative) theists insist it does. "

    Strawmen.


    Nothing to do with the strawman fallacy, little Anny -troll phony--but spout quasi-philosophical BS and some idiots around here might believe you. Many believers (including Feser at times) do use something like a Ad Logos argument..given human thinking/ Reason ->..G*d exists. Which is not all reasonable.

    ReplyDelete
  77. @Feser

    Yet another piece that demonstrates that, not that scientism is a problem, but that extreme philosophers think they are more important than they actually are.

    It is an ad hominem fallacy to claim in any way that the lack of intellectual sophistication on the part of "New Atheists" affects the strength of the IMPLICATIONS of their arguments, ie, religious philosophy is pointless.

    It's quite reasonable to conclude that science is the best way of studying the world given its many successes in both material and cultural life in a way no religion or "theoretical philosophy" has.

    Religion and philosophy of the intellectually sophisticated kind are removed from their stated purpose and prove nothing other than the fact that anyone can create their own "universal set" in which their beliefs hold and others' don't with absolutely no bearing a world that actually exists.

    Extreme abstract philosophy is already pointless. Adding religion to that is multiplying by 0.

    ReplyDelete
  78. @kwan_e

    There is little reason to take your response seriously.

    You accuse Dr. Feser of an ad hominem fallacy yet refer to him as an "extreme philosopher who thinks he is more important than he is"?

    So an argument ad hominem is Ok if you do it?

    Right! Sure pal!

    Plus you haven't really offered us any rational rebuttal. You are just nay-saying and name calling.

    I would forgive the name calling if you coupled it with a rational response.

    But we can't have everything.

    Ta! Taa!

    ReplyDelete